LSAT and Law School Admissions Forum

Get expert LSAT preparation and law school admissions advice from PowerScore Test Preparation.

 Administrator
PowerScore Staff
  • PowerScore Staff
  • Posts: 8917
  • Joined: Feb 02, 2011
|
#37023
Please post below with any questions!
 swong1267
  • Posts: 24
  • Joined: Nov 25, 2017
|
#43560
Can you explain why D is the correct answer?
 Emily Haney-Caron
PowerScore Staff
  • PowerScore Staff
  • Posts: 577
  • Joined: Jan 12, 2012
|
#43615
Hi swong1267,

This is a Must Be True question, so you're looking for the answer choice the definitely follows from the stimulus. The stimulus basically says most fad diets tell everyone to eat just a limited number of nutrients, but different people need different nutrients. It therefore follows that not everyone has their dietary needs met by those diets (because people need different things, and not all people will need just the limited things the particular fad diet recommends).

Hope that helps!
 gillwei
  • Posts: 2
  • Joined: Jun 20, 2018
|
#46773
Hello,
I am confused as to why the question is D and not C. The author states that "dietary needs vary wildly from person to person" and supports this by stating that different foods are needed to help treat/prevent different health problems. For the author's conclusion to be drawn, wouldn't the author first need to therefore establish that each person needs must have different health problems and dietary needs?

Thanks!
User avatar
 Jonathan Evans
PowerScore Staff
  • PowerScore Staff
  • Posts: 726
  • Joined: Jun 09, 2016
|
#46788
Hi, Gillwei,

Welcome to the forums! Excellent question.

This "Must Be True" question does lead the reader to reach an implicit conclusion, that fad diets fail to meet everyone's dietary needs; however, the author never states this conclusion outright. Instead, the information is presented more in the format of a fact set.

Our job is to use the information in the stimulus to prove a statement in the credited answer choice. We are not so much concerned with the author's conclusion (if there is one) as we are concerned with making our own valid inference.

Therefore, as you go through these answer choices, you are looking for a statement you can prove based on the stimulus. Let's take a look at answer choices (C) and (D):
  • Answer Choice (C) Can we prove that any two people have different health problems and thus different dietary needs? What does the stimulus say? The stimulus does include a statement similar to this starting with "But because different foods..." However, this statement is not exactly analogous to the statement in the answer choice. In the stimulus, we know that different people have different health problems, but we do not know that every single person has unique health issues that are different from everyone else's.

    In other words, the fact that "dietary needs vary widely from person to person" is not the same as saying "any two people have different dietary needs." The first statement implies that there are lots of different people with different needs. The second is a much stronger statement; the statement in this answer choice implies that every single person is unique.

    This kind of strong language ("any two people") is a common red flag for a trap answer. We do not have sufficient support for Answer Choice (C).

    Answer Choice (D) What do we know about "most fad diets?" We know that "most fad diets" prescribe a narrow range of foods. What do we know about people's dietary needs? We know they vary widely among people. Is there anything we can infer by combining the information in these two statements? Yes, we may infer correctly that most fad diets do not meet everyone's needs. We could phrase this idea also as "most fad diets fail to meet at least some people's needs." This is a close paraphrase of the statement in Answer Choice (D), and this is why (D) is the most strongly supported answer.

    Notice as well the weaker language about "some people's dietary needs." This kind of less extreme language is more common in credited responses to Must Be True questions.
I hope this helps!
 mcassidy1
  • Posts: 8
  • Joined: Sep 03, 2019
|
#67828
Why is answer B wrong?
 Rachael Wilkenfeld
PowerScore Staff
  • PowerScore Staff
  • Posts: 1358
  • Joined: Dec 15, 2011
|
#67836
Hi mcassidy1,

With this question type, we are looking for something that is provably true based on the information in the stimulus. When we look at answer choice (B) it states that fruits and vegetables are the only foods that can provide enough different nutrients to protect against lots of different health problems. Looking back to our stimulus, we can see that fruits and vegetables do provide enough of different nutrients. But do we see anywhere that it says they are the ONLY foods that do this?

You might think to yourself that fruits and vegetables were the only ones mentioned. But that doesn't mean they have to be the only ones that exist. Imagine you were talking to a friend about sunsets. Your friend tells you that San Francisco Bay has amazing sunsets where you can see beautiful colors all the way to the horizon. That doesn't mean that San Francisco is the only place to see amazing sunsets. Saying that one place has lovely sunsets doesn't exclude other places having them. Saying that one type of food has certain properties doesn't exclude other types of food having them too.

Hope that helps!
Rachael
User avatar
 Canada101
  • Posts: 6
  • Joined: Aug 31, 2023
|
#104189
Hello, I hope you are all well!

Could you please explain why answer A is incorrect? I believe I understand why but I am not sure if my line of thinking is correct.

This is a Must Be True (MBT) Question which means it must pass the fact test. The correct answer choice will be proved by the stimulus (Family 1: Prove). If most fad diets prescribe a single range of nutrients how could they include fruits and vegetables who have a wide range of nutrients (the stimulus says they protect a wide range of health problems so they contain alot of nutrients?). So if most fad diets did include fruits and vegetables they would not be considered to prescribe a single range of nutrients - correct? Would this be considered an opposite answer of sorts?

Thank you for all your help!
User avatar
 Canada101
  • Posts: 6
  • Joined: Aug 31, 2023
|
#104190
Hello!

Could you please explain what is wrong with answer E (Why is it the wrong answer)?

Thank you!
User avatar
 Jeff Wren
PowerScore Staff
  • PowerScore Staff
  • Posts: 389
  • Joined: Oct 19, 2022
|
#104195
Hi Canada101,

Often wrong answers in Must Be True questions incorrectly mix different ideas mentioned in the stimulus to say something that was not stated or implied in the stimulus.

Answer A is an example of this. Answer A mixes the idea of "most fad diets" discussed in the first sentence and "eating plenty of fruits and vegetables" in the third sentence.

I agree that Answer A does seem to be saying the opposite of what can be inferred from the stimulus, but even if it were just unsupported/unknown, it would still be incorrect.

Answer E is discussing foods "other than fruits and vegetables." Based on the stimulus, we don't know whether or not there are very few nutrients contained in every food other than fruits and vegetables. You don't want to equate "a single narrow range of nutrients" (my emphasis) mentioned in the stimulus to "very few nutrients" in Answer E.

For example, based on the stimulus, there could be more than a very few nutrients contained in every food other than fruits in vegetables while still falling in a narrow range of nutrients.

Get the most out of your LSAT Prep Plus subscription.

Analyze and track your performance with our Testing and Analytics Package.